Compute the flux of a vector field through the boundary of a solid

Click For Summary
The discussion focuses on using the Gauss divergence theorem to compute the flux of a vector field through a solid's boundary. The divergence of the vector field is correctly identified as div F = 2z, but the parametrization using cylindrical coordinates is questioned due to incorrect volume bounds. The correct volume is described as lying above the paraboloid z = x² + y² and below z = 4, with the appropriate limits for integration in cylindrical coordinates being r from 0 to 2, θ from 0 to 2π, and z from r² to 4. The initial bounds provided do not accurately represent the solid's volume, leading to an incorrect result. Properly adjusting the integration limits is essential for an accurate computation of the flux.
DottZakapa
Messages
239
Reaction score
17
Homework Statement
Compute the outward flux of the vector field F(x,y,z) = 2x,−2y,z2 through the boundary of the solid
Ω= (x,y,z)∈R3: x2+y2≤z≤4 .
Relevant Equations
flux through a surface
is it correct if i use Gauss divergence theorem, computing the divergence of the vector filed,
that is :

div F =2z
then parametrising with cylindrical coordinates
##x=rcos\alpha##
##y=rsin\alpha##
z=t

1≤r≤2
0≤##\theta##≤2π
0≤t≤4

##\int_{0}^{2\pi} \int_{0}^{2} \int_{0}^{4} 2tr \, dt \, dr \,d\theta##

but i guess there is something missing because the result is not correct
 
Physics news on Phys.org
DottZakapa said:
1≤r≤2
0≤θθ\theta≤2π
0≤t≤4
This describes a hollowed out cylinder, which is not your volume. What is your reasoning behind theses bounds?
 
z= x^2+ y^2 is a paraboloid. The volume lies above that paraboloid and below z= 4. Of course, x^2+ y^2= 4 is the circle in the x,y plane with center at the origin and radius 2.

In cartesian coordinates, the integral of any function, f(x, y, z) over that region would be \int_{x= -2}^2 \int_{y= -\sqrt{4- x^2}}^{\sqrt{4- x^2}}\int_{z= x^2+ y^2}^4 f(x,y,z)dzdydx.

In cylindrical coordinates, \int_{r= 0}^2\int_{\theta= 0}^{2\pi}\int_{z= r^2}^4 f(r,\theta,z) dzd\theta dr.
 
Question: A clock's minute hand has length 4 and its hour hand has length 3. What is the distance between the tips at the moment when it is increasing most rapidly?(Putnam Exam Question) Answer: Making assumption that both the hands moves at constant angular velocities, the answer is ## \sqrt{7} .## But don't you think this assumption is somewhat doubtful and wrong?

Similar threads

Replies
6
Views
2K
  • · Replies 6 ·
Replies
6
Views
2K
Replies
8
Views
3K
  • · Replies 7 ·
Replies
7
Views
2K
  • · Replies 1 ·
Replies
1
Views
1K
  • · Replies 4 ·
Replies
4
Views
2K
  • · Replies 2 ·
Replies
2
Views
2K
  • · Replies 8 ·
Replies
8
Views
2K
Replies
5
Views
1K
  • · Replies 3 ·
Replies
3
Views
2K